0% found this document useful (0 votes)
50 views

MTH 304 Final Solutions

This document contains solutions to problems from an MTH 304 final exam. The problems cover topics such as topological groups, one-point compactifications, and homeomorphisms between quotient spaces and tori. Specifically, it shows that GL(2,R) and SL(2,R) are topological groups, characterizes the one-point compactification of the natural numbers, and proves that a map induced by a matrix in SL(2,Z) descends to a homeomorphism of the torus.
Copyright
© © All Rights Reserved
We take content rights seriously. If you suspect this is your content, claim it here.
Available Formats
Download as PDF, TXT or read online on Scribd
0% found this document useful (0 votes)
50 views

MTH 304 Final Solutions

This document contains solutions to problems from an MTH 304 final exam. The problems cover topics such as topological groups, one-point compactifications, and homeomorphisms between quotient spaces and tori. Specifically, it shows that GL(2,R) and SL(2,R) are topological groups, characterizes the one-point compactification of the natural numbers, and proves that a map induced by a matrix in SL(2,Z) descends to a homeomorphism of the torus.
Copyright
© © All Rights Reserved
We take content rights seriously. If you suspect this is your content, claim it here.
Available Formats
Download as PDF, TXT or read online on Scribd
You are on page 1/ 7

MTH 304 Final Solutions

1. Consider the multiplicative matrix groups


  
a b 4
GL(2, R) = | (a, b, c, d) ∈ R and ad − bc 6= 0
c d

and
  
a b 4
SL(2, R) = | (a, b, c, d) ∈ R and ad − bc = 1 .
c d

(a) Show that GL(2, R) and SL(2, R) are topological groups.


(b) Show that GL(2, R) is homeomorphic to an open subspace of R4 ,
while SL(2, R) is homeomorphic to a closed subspace of R4 . [Hint:
Consider the determinant map Det : R4 → R.]
(c) Show that GL(2, R) is not connected, and SL(2, R) is noncom-
pact.

Solution. (a) As both G and H are subsets of R4 , they inherit the


subspace topology from the standard topology in the ambient space.
We know from MTH 301 that both G = GL(2, R) and H = SL(2, R)
are multiplicative groups, and H E G. So it suffices to show that G
is a topological group. In other words, we need to establish that the
matrix product operation
ϕ
ϕ : G × G → G : (A, B) 7−
→ AB,

and the matrix inversion operation


I
7− A−1
I:G→G : A→

are continuous maps. We know that for two matrices


   
a1 b1 a2 b2
A= ,B= ∈ G,
c1 d1 c2 d2

we have
 
a1 a2 + b1 c2 a1 b2 + b1 d2
ϕ(A, B) = AB = ,
c1 a2 + d1 c2 c1 b2 + d1 d2

1
 
a b
and for C = ∈ G, we have
c d
 
−1 1 d −b
I(C) = C = .
ad − bc −c a

Viewing ϕ as a map
ϕ
R4 × R4 → R4 : (A, B) 7−
→ ((AB)11 , (AB)12 , (AB)21 , (AB)22 ),
(where A = (a1 , b1 , c1 , d1 ), B = (a2 , b2 , c2 , d2 ) ∈ R4 ) we see that its
four component functions of the form (A, B) 7→ (AB)ij are multivari-
able polynomials, and hence ϕ is continuous. Similarly, viewing I as a
function R4 → R4 , we see that its four components functions are ratio-
nal functions (i.e. polynomial/polynomial), which have the a common
denominator ad − bc 6= 0. Hence, I is a continuous function, and this
shows that G is a topological group.
(b) Now consider the determinant map
 
a b Det
Det : G → R : 7−−→ ad − bc.
c d
Once again, viewing Det as a map R4 → R, we see that as ad − bc is
a polynomial, and so Det is continuous. It is apparent that as R is a
T1 space,
A = Det−1 (R \ {0}) and Det−1 ({1}),
from which (b) follows.
(c) The disconnectedness of G follows from the fact that the open
subsets
Det−1 ((−∞, 0)) and Det−1 ((0, ∞))
form a separation for G. To see the noncompactness of H, it suffices
to show that H is unbounded under the standard metric in R4 (by the
Heine-Borel property). For any n ∈ N, consider the matrix An ∈ H
defined by  
n 0
An = .
0 1/n
p
Then kAn k = n2 + 1/n2 , and
lim kAn k = ∞,
n→∞

which shows that {An | n ∈ N} is an unbounded subset of H. As H


has an unbounded subset, it is unbounded.

2
2. Show that if X is separable, then every collection of disjoint open sets
in X is countable.
Solution. Let A be a countable dense subset of X such that Ā = X.
Let {Uα }α∈J be an arbitrary collection of disjoint open sets in X.
Since Ā = X, for each α, there exists an xα ∈ A ∩ Uα . Moreover,
the fact that the open sets in {Uα }α∈J are mutually disjoint implies
that xα 6= xβ , whenever α 6= β. As A is countable, Y = {xα | α ∈ J}
is a countable subset of X. Since Y is bijective with J, J has to be
countable.
3. (a) Define the one-point compactification of a locally compact Haus-
dorff space.
(b) Show that the open point compactification of N is homeomorphic
to {1/n | n ∈ N} ∪ {0}.
Solution. (a) See 1.2 (xxx) in the Lesson Plan.
(b) Let K = {1/n | n ∈ N}. The inversion map
ι
ι : R \ {0} → R \ {0} : x →
7− 1/x
is a homeomorphism, as it is a rational function. This shows that
ι|N : N → K
is a homeomorphism. As K is a closed subset of the locally compact
Hausdorff space R, K is locally compact. Hence, it follows from 1.2
(xxxi) that K has a one-point compacitification. But that fact that
K = K ∪ {0} is compact space (being a closed and bounded subspace
of R) and Hausdorff implies that K is the unique one point compact-
ification of K, up to homeomorphism.
Let X ∗ = N ∪ {∞} be the one-point compactification of X = N. Then
by defining ι(∞) = 0, the map ι extends to a bijective map
ι̂ : X ∗ → Y,
where Y = K ∪ {0}. Since X ∗ compact and Y is Hausdorff, it suffices
to show that ι̂ is continuous, and in particular, ι̂ is continuous at ∞.
Let U be a neighborhood of 0 in Y . Then by definition Y \ U is
compact, which implies that ι̂−1 (Y \ U ) = ι−1 (K \ U ) is compact, and
so
ι̂−1 (U ) = X \ ι̂−1 (Y \ U )
is a open neighborhood of ∞, which is mapped into U . This shows
that ι̂ is continuous, and hence a homeomorphism.

3
4. Let X be a nonempty compact Hausdorff space without isolated points.

(a) Show that for each nonempty open U ⊂ X, and x ∈ X, there


exists an open set V ⊂ U such that x ∈
/ V.
(b) Show that there exists no surjective map f : N → X. [Hint:
Consider x1 = f (1) and U = X and apply (a) to get a V . Now
take x2 = f (2) and U = V , and so on. Finally, use the finite
intersection property.]

Solution 1. The argument is analogous to the one used in the proof


of Theorem 27.7 (Page 174) in Munkres.

5. If every real-valued continuous function on a metric space X is bounded,


then show that X is compact. [Hint: If X is not compact, consider
a sequence (xn ) with no covergent subsequence. Consider the map
xn 7→ n, and use the Tietze extension theorem.]
Solution. We know that a metric space is compact iff its sequentially
compact. Suppose that X is not compact. Then X is not sequentially
compact, which implies that there exists a sequence (xn ) in X that has
no convergent subsequence. This implies that the set A = {xn | n ∈ N}
has no limit points, and so A is closed in X. Define a function
f
f : A → N(⊂ R) : xn 7−
→ n.

Then clearly, f is a continuous, as its a bijective map between two


discrete subspaces. Moreover, as X is metrizable, it is normal, and by
the Tietze’s extension theorem, f extends to a continuous map

fˆ : X → R,

which is clearly unbounded.

6. Consider the standard quotient map q : R2 → S 1 × S 1 (≈ R2 /Z2 )


induced by the equivalence relation ∼ on R2 defined by

(x1 , y1 ) ∼ (x2 , y2 ) ⇐⇒ (x2 − x1 , y2 − y1 ) ∈ Z2 .

Consider the multiplicative matrix group


  
a b 4
SL(2, Z) = | (a, b, c, d) ∈ Z and ad − bc = 1 .
c d

4
 
a b
For a fixed matrix A = ∈ SL(2, Z), define a map MA : R2 →
c d
R2 given by
    
a b x ax + by
MA ((x, y)) = = ,
c d y cx + dy

for any (x, y) ∈ R2 .

(a) Show that MA induces a map M fA : S 1 × S 1 → S 1 × S 1 , which is


a homeomorphism.
(b) Show that for a fixed A ∈ SL(2, Z), we have q ◦ MA = M fA ◦ q.
Solution. (a) & (b) We know that by definition, q((x, y)) =
[(x, y)], where [(x, y)] denotes the equivalence class of (x, y) under
∼, given by

[(x, y)] = {(x + p, y + q) | p, q ∈ Z}.

We define
fA : S 1 × S 1 → S 1 × S 1
M
by
fA ([(x, y)]) = [MA ((x, y))], for all [(x, y)] ∈ S 1 × S 1 .
M

Then M fA is well-defined, for if [(x, y)] = [(x0 , y 0 )], then there


exists p, q ∈ Z such that (x + p, y + q) = (x0 , y 0 ), so that

fA ([(x0 , y 0 )]) =
M [MA (x0 , y 0 )]
= [MA (x + p, y + q)]
= [a(x + p) + b(y + q), c(x + p) + d(y + q)]
= [MA (x, y) + MA (p, q)]
= [MA (x, y)]
= M
fA ([(x, y)].

Moreover, for all (x, y) ∈ R2 , we have that

(q ◦ MA )((x, y)) = q(MA ((x, y))


= [MA ((x, y))]
= M
fA ([x, y])
= M
fA (q((x, y))
= fA ◦ q)((x, y)).
(M

5
This shows that the diagram
M
R2 A
−−−−→ R2
 
p
y
p
y (*)
M
S 1 × S 1 −−−−
A
→ S1 × S1
f

is commutative.
Since A ∈ SL(2, Z), MA is an invertible linear map, hence a
homeomorphism. Now for any [(x0 , y 0 )] ∈ S 1 × S 1 , there exits
(x, y) ∈ R2 such that MA ((x, y)) = (x0 , y 0 ), which implies that
fA ([(x, y)]) = [MA ((x, y))] = [(x0 , y 0 )],
M

and so it follows that M


fA is surjective. Moreover, we have that

M fA ([(x0 , y 0 )])
fA ([(x, y)]) = M =⇒ [MA ((x, y))] = [MA ((x0 , y 0 ))]
=⇒ (MfA ◦ q)((x, y)) = (M fA ◦ q)((x0 , y 0 ))
=⇒ (q ◦ MA )((x, y)) = (q ◦ MA )((x0 , y 0 ))
=⇒ [MA ((x, y))] = [MA ((x0 , y 0 ))]
=⇒ [(x, y)] = [(x0 , y 0 )],

from which the injectivity of M fA follows.


Since q is a open and continuous map (why?), for an open set U ⊂
S 1 × S 1 , we have that q −1 (U ) is open in R2 , so that MA (q −1 (U )
is open in R2 , and so

(q ◦ MA ◦ q −1 )(U ) = M
fA (U )

is open in S 1 × S 1 . Finally, for an open set V in S 1 × S 1 , we have


that
f−1 (V ) = (q ◦ MA ◦ q −1 )−1 (V ) = (q ◦ M −1 q −1 )(V ),
M A A

which is open in S 1 × S 1 . This shows that M


fA is a homeomor-
phism.
Solution 2. Alternatively, one could consider the diagram

R2 (**)
q◦MA
q
 &
M
/ S1 × S1
fA
S1 × S1

6
that is equivalent to the diagram (*) above. As the map q ◦ MA
is constant on each fiber of q (why?), by 1.10 (x) of the Lesson
Plan, there exists a map M fA as indicated in (**), that makes
the diagram commute. Consequently, (*) is also commutative.
Moreover, since q and MA are continuous maps, we have that
q ◦ MA is continuous, and once again, 1.10 (x) would imply that
M
fA is continuous.
The fact that q is an open map (from class) and MA is a homeo-
morphism implies that q◦MA is a surjective, continuous and open
map, and hence a quotient map. Finally, by 1.10 (xi), q ◦ MA will
induce a homeomorphism S 1 × S 1 → S 1 × S 1 , which in this case
is precisely the map MfA .

You might also like